Spooky + scary = Summation

Algebra Level 5

If S k , ( k = 1 , 2 , 3 , 100 ) , S_{k}, (k=1,2,3 \cdots ,100), denotes the infinite geometric progression sum , whose first term is k 1 k ! \dfrac{k - 1}{k!} and common ratio is 1 k \dfrac{1}{k} , then find ( 100 ) 2 100 ! + k = 2 100 ( k 2 3 k + 1 ) S k \large \dfrac{(100)^{2}}{100!} + \displaystyle \sum_{k=2}^{100} | (k^{2} - 3 k + 1) \cdot S_{k} |


The answer is 3.

This section requires Javascript.
You are seeing this because something didn't load right. We suggest you, (a) try refreshing the page, (b) enabling javascript if it is disabled on your browser and, finally, (c) loading the non-javascript version of this page . We're sorry about the hassle.

1 solution

Rishabh Jain
Mar 17, 2016

Just loved solving this question @Mohit Gupta -Do you design them on your own or are they from a book??

Using formula for infinite GP:- S k = k 1 k ! 1 1 k = 1 ( k 1 ) ! S_k=\dfrac{\frac{k-1}{k!}}{1-\frac 1k}=\dfrac{1}{(k-1)!}

Let T = k = 2 100 ( k 2 3 k + 1 ) S k \large\text{Let } \mathfrak{T}= \displaystyle \sum_{k=2}^{100} | (k^{2} - 3 k + 1) \cdot S_{k} | = k = 2 100 ( k 2 3 k + 1 ) ( k 1 ) ! \large =\displaystyle \sum_{k=2}^{100} |\dfrac{ (k^{2} - 3 k + 1) } {(k-1)!}|

= k = 2 100 ( k 1 ) ( k 2 ) 1 ( k 1 ) ! \large =\displaystyle \sum_{k=2}^{100} |\dfrac{(k-1)(k-2)-1}{(k-1)!}|

= 1 + k = 3 100 1 ( k 3 ) ! 1 ( k 1 ) ! W \large=|-1|+\underbrace{\large \displaystyle \sum_{k=3}^{100} |\dfrac{1}{(k-3)!}-\dfrac{1}{(k-1)!}|}_{\mathfrak{W}}

W = ( ( 1 1 2 ! ) + ( 1 1 3 ! ) + ( 1 2 ! 1 4 ! ) + ( 1 96 ! 1 98 ! ) + ( 1 97 ! 1 99 ! ) ) \Large \mathfrak{W}=\left( \begin{aligned} & ~~~\left(1-\cancel{\color{#3D99F6}{\dfrac{1}{2!}}}\right)\\&+\left(1-\cancel{\color{#D61F06}{ \dfrac{1}{3!}}}\right)\\&+\left(\cancel{\color{#3D99F6}{\dfrac{1}{2!}}}-\cancel{\color{#EC7300}{\dfrac{1}{4!}}}\right) \\&\cdots\\&\cdots\\&\cdots\\&+\left(\cancel{\color{#20A900}{\dfrac{1}{96!}}}-\dfrac{1}{98!}\right)\\&+\left(\cancel{\color{#624F41}{\dfrac{1}{97!}}}-\dfrac{1}{99!}\right)\end{aligned} \right) A T e l e s c o p i c S e r i e s \color{#302B94}{\mathbf{A ~Telescopic ~Series}}

= 2 1 98 ! 1 99 ! = 2 10 0 2 100 ! \large=2-\dfrac{1}{98!}-\dfrac{1}{99!}=2-\dfrac{100^2}{100!}

Therefore required sum:- 10 0 2 100 ! + 1 + W \large\dfrac{100^2}{100!}+1+\mathfrak{W}

= 3 \Huge=\color{#456461}{\boxed{\color{#EC7300}{\boxed{\color{#0C6AC7}{\mathbf{3}}}}}}

No actually all these question are from different sources...the ones which i feel are cool so i modify them to harder level and then post on brilliant.....For example my this question Generalisations make things better was initially a easy question so i thought why not generalise it for all polynomials......other one Double refraction or trouble refraction ...was an easy question from INJSO....so i just post the problems which i think are worth sharing and more because of my curiosity to know that is a better soln. than mine possible for it :)

Mohit Gupta - 5 years, 3 months ago

Log in to reply

Nice.... So how did you solved this one?

Rishabh Jain - 5 years, 3 months ago

Log in to reply

Exactly like the one you solved.....however it took me just a bit time to discover that it wad a telescoping series.....

Please can you help me with this question x^{2} + \frac{x^{2}}{(x+1)^{2} = 3 .

Plz tell can we solve this question through algebraic manipulations...or some other comcept.

Mohit Gupta - 5 years, 3 months ago

Log in to reply

@Mohit Gupta I think you're saying : x 2 + x 2 ( x + 1 ) 2 = 3 x^2+\dfrac{x^2}{(x+1)^2}=3 Make the substitution x + 1 = t x+1=t . ( t 1 ) 2 + ( t 1 ) 2 t 2 = 3 \implies (t-1)^2+\dfrac{(t-1)^2}{t^2}=3 t 2 2 t + 1 t 2 2 t = 1 \implies t^2-2t+\dfrac{1}{t^2}-\dfrac{2}{t}=1 t 2 + 1 t 2 2 ( t + 1 t ) = 1 \implies t^2+\dfrac{1}{t^2}-2(t+\dfrac 1t)=1 ( t + 1 t ) 2 2 ( t + 1 t ) = 3 (t+\dfrac 1t)^2-2(t+\dfrac 1t)=3 Again substitute t + 1 t = p t+\dfrac 1t=p to get a quadratic in p p . p 2 2 p 3 = 0 p^2-2p-3=0 ( p 3 ) ( p + 2 ) = 0 (p-3)(p+2)=0 Following process is easy now I think.... :-)

Rishabh Jain - 5 years, 3 months ago

Log in to reply

@Rishabh Jain one thing.....in last eqn. you copied 1 as 3...

Mohit Gupta - 5 years, 3 months ago

Log in to reply

@Mohit Gupta No .. We have to add 2 on both the sides to transform t 2 + 1 t 2 t^2+\dfrac{1}{t^2} to ( t + 1 t ) 2 (t+\dfrac 1t)^2 .. Isn't it??

Rishabh Jain - 5 years, 3 months ago

Log in to reply

@Rishabh Jain Ohhhhh.....i just forgot about that......thanks understood whete i was getting it wrong.....really a foolish mistake by me :P

Mohit Gupta - 5 years, 3 months ago

Log in to reply

@Mohit Gupta No problem.. :-) Keep giving beautiful problems...

Rishabh Jain - 5 years, 3 months ago

Log in to reply

@Rishabh Jain Same problem as posted by mudit bansal, under the name 'Cool Summation'

A Former Brilliant Member - 5 years, 2 months ago

Log in to reply

@A Former Brilliant Member Ohk.. Thanks Might be this problem... But I solved mohit's problem first time and I loved it... Its a bit tricky..

Rishabh Jain - 5 years, 2 months ago

Nice solution!

Akshat Sharda - 5 years, 2 months ago

Log in to reply

Thanks.. :-)

Rishabh Jain - 5 years, 2 months ago

Nice solution. I don't remember the year, but this is a JEE problem.

Saurabh Chaturvedi - 5 years, 2 months ago

Log in to reply

Thanks... It would be really grateful if you could tell the year or maybe guess around which year this problem was asked in JEE...

Rishabh Jain - 5 years, 2 months ago

JEE 2010

!!

Ravi Dwivedi - 5 years, 2 months ago

Log in to reply

Yup... Thanks for that information due to which I managed to find it now..... JEE-10 Paper-1 Q-28.

Rishabh Jain - 5 years, 2 months ago

Log in to reply

2010 mathematics paper was actually very good I think

Ravi Dwivedi - 5 years, 2 months ago

Log in to reply

@Ravi Dwivedi Right ..... Absolutely .....

Rishabh Jain - 5 years, 2 months ago

And there is no Q-28, it depends on the set you get

Ravi Dwivedi - 5 years, 2 months ago

Log in to reply

@Ravi Dwivedi Ohk my bad... I had KD joshi commentaries so I couldn't find the set..

Rishabh Jain - 5 years, 2 months ago

0 pending reports

×

Problem Loading...

Note Loading...

Set Loading...